24th IMO 1983 shortlist

------
 
 
Problem 20

Find the greatest integer not exceeding 1 + 1/2k + 1/3k + ... + 1/Nk, where k = 1982/1983 and N = 21983.

 

Solution

We have (a1983 - b1983) = (a - b)(a1982 + a1981b + a1980b2 + ... + b1982). So if a > b, we have (a1983 - b1983) < (a - b) 1983 a1982. Take a = m1/1983, b = (m-1)1/1983 and this becomes (1/m)1982/1983 < 1983 (m1/1983 - (m-1)1/1983). Summing we get 1/2k + 1/3k + ... + 1/Nk < 1983( N1/1983 - 1) = 1983 (2 - 1). So 1 + 1/2k + 1/3k + ... + 1/Nk < 1984.

Similarly, (a1983 - b1983) > (a - b) 1983 b1982, so (1/(m-1))k > 1983 (m1/1983 - (m-1)1/1983). Summing gives 1 + 1/2k + 1/3k + ... + 1/(N-1)k > 1983(2 - 1) = 1983. Hence 1 + 1/2k + 1/3k + ... + 1/Nk lies strictly between 1983 and 1984. So the integer part is 1983.

 


 

24th IMO shortlist 1983

© John Scholes
jscholes@kalva.demon.co.uk
26 Nov 2003
Last corrected/updated 26 Nov 03